From: K_h on

"Virgil" <Virgil(a)home.esc> wrote in message
news:Virgil-1E8B09.00355309122009(a)newsfarm.iad.highwinds-media.com...
> In article
> <c8idnc6JG6hv34LWnZ2dnUVZ_q-dnZ2d(a)giganews.com>,
> "K_h" <KHolmes(a)SX729.com> wrote:
>
>> It should be pointed out that N is a limit set even if N
>> is
>> initially given by a definition that doesn't involve the
>> notion of a limit.
>
>
> The issue between Dik and WM is whether the limit of a
> sequence of sets
> according to Dik's definition of such limits is
> necessarily the same as
> the limit of the sequence of cardinalities for those sets.
>
> And Dik quire successfully gave an example in which the
> limits differ.

I suspect those definitions are not valid. The definition I
used is the one on wikipedia and is generally `standard' --
as I've seen it in numerous places, including books and
websites. That definition is below. Using this definition
it is possible to prove that N is a limit and I presented a
proof of this in my previous post.

Let /\ = Intersection
Let \/ = union
Define infimum and supremum as follows:
liminf X_n=\/(n=0, oo)[/\(m=n, oo) X_m]
(n-->oo)
limsup X_n=/\(n=0, oo)[\/(m=n, oo) X_m]
(n-->oo)

> Here is another way to see that N is a
>> limit even if you consider it bad taste to define it in
>> those terms:
>>
>> Let \/ = union
>>
>> Let /\ = Intersection
>>
>> Define infimum and supremum as follows:
>>
>> liminf X_n=\/(n=0, oo)[/\(m=n, oo) X_m]
>> (n-->oo)
>>
>> limsup X_n=/\(n=0, oo)[\/(m=n, oo) X_m]
>> (n-->oo)
>>
>> If these two are the same then the limit exists and is
>> both
>> of them.
>
> The issue is not whether the naturals are such a limit but
> whether for
> every so defined limit the cardinality of the limit equals
> the limit
> cardinality of their cardinalities, which is different
> sort of limit.

It doesn't sound like a valid limit. With the definitions I
provided it is possible to prove, in your words, that "the
cardinality of the limit equals the cardinality of their
cardinalities". It is actually intuitively obvious: the set
of finite cardinals 0,1,2,3,... and the set of finite
ordinals 0,1,2,3,... both have cardinality ALEPH_0.

k


From: K_h on

"WM" <mueckenh(a)rz.fh-augsburg.de> wrote in message
news:b6fe23bb-f753-4cfc-a9d6-79711f3a1e5d(a)r40g2000yqn.googlegroups.com...
> On 9 Dez., 08:35, Virgil <Vir...(a)home.esc> wrote:
>> In article
>> <c8idnc6JG6hv34LWnZ2dnUVZ_q-dn...(a)giganews.com>,
>
>
>> However, in the discussion between Dik and WM, Dik gave
>> SPECIFIC
>> definition of what HE meant by the limit of a seqeunce of
>> sets which
>> differed from that in your citation.
>
> Dik:
> Given a sequence of sets S_n then:
> lim sup{n -> oo} S_n contains those elements that occur
> in
> infinitely
> many S_n

I don't think this is a good definition for the limit of a
sequence of sets S_n. For instance, consider the
alternating sets:

S_0 = {0, 2, 4, 6, 8,...}
S_1 = {-1, -3, -5, ...}
S_2 = {0, 2, 4, 6, 8,...}
S_3 = {-1, -3, -5, ...}
S_4 = {0, 2, 4, 6, 8,...}
S_5 = {-1, -3, -5, ...}
....

To me, I don't see the S_n converging to anything, certainly
not the set of negative odd numbers union the set of even
numbers. Like Thompson's lamp alternating between on and
off it reaches no limit in any intuitive sense of what a
limit is.

> lim inf{n -> oo} S_n contains those elements that occur
> in all S_n
> from
> a certain S_n (which can be different
> for each
> element).
> lim{n -> oo} S_n exists whenever lim sup and lim inf are
> equal.
>>
>> > Define infimum and supremum as follows:
>>
>> > liminf X_n=\/(n=0, oo)[/\(m=n, oo) X_m]
>> > (n-->oo)
>>
>> > limsup X_n=/\(n=0, oo)[\/(m=n, oo) X_m]
>> > (n-->oo)
>>
>> > If these two are the same then the limit exists and is
>> > both
>> > of them.
>>
>> The issue is not whether the naturals are such a limit
>> but whether for
>> every so defined limit the cardinality of the limit
>> equals the limit
>> cardinality of their cardinalities, which is different
>> sort of limit
>
> If the limit set emerges continuously (i.e. growing
> one-by-one) from
> the sets of the sequence, then the cardinality will be
> bound to this
> process and will also continuously emerge from the
> cardinalities of
> the sets of the sequence.
>
> If there is a gap in one sequence and no gap in the other,
> then this
> indicates that at least one of the limits does not exist,
> probably
> both do not exist.
>
> This has been substanciated by the following
> thought-experiment:
> If N is generated as a limit, then it is generated by
> adding number
> after number.
> When using an intermediate reservoir, as shown in my
> lesson
> http://www.hs-augsburg.de/~mueckenh/GU/GU12.PPT#394,22,Folie
> 22
> it becomes clear that N cannot be generated by adding
> number after
> number.

Why not? Say we have an infinitely large sheet of paper and
we print each natural number, n, on the paper at time
t=1-1/(n+1). Certainly at time t=1 we have all the natruals
printed on the page.

> Therefore neither N nor Card(N) are meaningful concepts -
> at least not
> in mathematics.

k


From: Virgil on
In article <QP-dnV0EIYPt2b3WnZ2dnUVZ_h2dnZ2d(a)giganews.com>,
"K_h" <KHolmes(a)SX729.com> wrote:

> "Virgil" <Virgil(a)home.esc> wrote in message
> news:Virgil-1E8B09.00355309122009(a)newsfarm.iad.highwinds-media.com...
> > In article
> > <c8idnc6JG6hv34LWnZ2dnUVZ_q-dnZ2d(a)giganews.com>,
> > "K_h" <KHolmes(a)SX729.com> wrote:
> >
> >> It should be pointed out that N is a limit set even if N
> >> is
> >> initially given by a definition that doesn't involve the
> >> notion of a limit.
> >
> >
> > The issue between Dik and WM is whether the limit of a
> > sequence of sets
> > according to Dik's definition of such limits is
> > necessarily the same as
> > the limit of the sequence of cardinalities for those sets.
> >
> > And Dik quire successfully gave an example in which the
> > limits differ.
>
> I suspect those definitions are not valid.

Irrelevant to whether that definition "is valid" or corresponds to
anyone else's. That is the definition under which WM claimed the two
limit processes coincide, but they do not.
From: WM on
On 10 Dez., 01:58, "K_h" <KHol...(a)SX729.com> wrote:

> > When using an intermediate reservoir, as shown in my
> > lesson
> >http://www.hs-augsburg.de/~mueckenh/GU/GU12.PPT#394,22,Folie
> > 22
> > it becomes clear that N cannot be generated by adding
> > number after
> > number.
>
> Why not?  Say we have an infinitely large sheet of paper and
> we print each natural number, n, on the paper at time
> t=1-1/(n+1).  Certainly at time t=1 we have all the naturals
> printed on the page.

It seems so. But it is wrong. You see it if you consider the
alternative process using an intermediate reservoir as "realized" in
my lesson above.

naturals - reservoir - paper
N - { } - { }
N/{1} - {1} - { }
N/{1,2} - {2} - {1}
N/{1,2,3} - {3} - {1,2}
....
N/{1,2,3, ...,n} - {n} - {1,2,3, ...,n-1}
....

The set in the middle contains a number at every time after t = 0.
Hence this number cannot yet have been printed on the paper (because
it will be printed only after its follower will have entered the
reservoir).

Regards, WM
From: Virgil on
In article
<b963b9aa-c345-43cf-bf78-e9e27401f539(a)c34g2000yqn.googlegroups.com>,
WM <mueckenh(a)rz.fh-augsburg.de> wrote:

> On 10 Dez., 01:58, "K_h" <KHol...(a)SX729.com> wrote:
>
> > > When using an intermediate reservoir, as shown in my
> > > lesson
> > >http://www.hs-augsburg.de/~mueckenh/GU/GU12.PPT#394,22,Folie
> > > 22
> > > it becomes clear that N cannot be generated by adding
> > > number after
> > > number.
> >
> > Why not? �Say we have an infinitely large sheet of paper and
> > we print each natural number, n, on the paper at time
> > t=1-1/(n+1). �Certainly at time t=1 we have all the naturals
> > printed on the page.
>
> It seems so. But it is wrong. You see it if you consider the
> alternative process using an intermediate reservoir as "realized" in
> my lesson above.
>
> naturals - reservoir - paper
> N - { } - { }
> N/{1} - {1} - { }
> N/{1,2} - {2} - {1}
> N/{1,2,3} - {3} - {1,2}
> ...
> N/{1,2,3, ...,n} - {n} - {1,2,3, ...,n-1}
> ...
>
> The set in the middle contains a number at every time after t = 0.
> Hence this number cannot yet have been printed on the paper (because
> it will be printed only after its follower will have entered the
> reservoir).

And when all the numbers have passed through your "reservoir", both into
and out ofd it, as will have happened by t = 1, which numbers does WM
claim will still be unprinted.

When going through the terms of an infinite sequence, as in the above,
EITHER the process hangs up on a particular term of that sequence OR it
goes through every term of the sequence, TERTIUM NON DATUR.